Những câu hỏi liên quan
hotboy
Xem chi tiết
Thám Tử THCS Nguyễn Hiếu
20 tháng 9 2015 lúc 16:26

Ta có: S=22+42+62+...+202

            =(2.1)2+(2.2)2+(2.3)2+...+(2.10)2

            =22.12+22.22+22.32+...+22.102

            =22.(1+22+32+...+102)

Mà 12+22+32+...+102=385 nên:

S=22.385

  =4.385

  =1540

Vậy S=1540

Bình luận (0)
thaophamthao
Xem chi tiết
hoabinhyenlang
9 tháng 4 2015 lúc 12:20

 

2^2=4=4.1^2

             4^2=16=4.2^2

             6^2=36=4.3^2

...

 20^2=400=4.10^2

nên: S=2^2+4^2+6^2+...+20^2=4.(1^2+2^2+3^2+...+10^2)

                                         =4.385

                                         =1540

  tổng S = 1540

Bình luận (0)
hoabinhyenlang
9 tháng 4 2015 lúc 12:21

tuy bài này không do mình giải nhưng bạn có thể yên tâm vì câu trả lời rất đúng

Bình luận (0)
๖ACE✪Hoàngミ★Việtツ
18 tháng 8 2017 lúc 8:16

2^2=4=4.1^2

             4^2=16=4.2^2

             6^2=36=4.3^2

...

 20^2=400=4.10^2

nên: S=2^2+4^2+6^2+...+20^2=4.(1^2+2^2+3^2+...+10^2)

                                         =4.385

                                         =1540

  tổng S = 1540

~~~~~~~~~~~ai đi ngang qua nhớ để lại k ~~~~~~~~~~~~~

 ~~~~~~~~~~~~ Chúc bạn sớm kiếm được nhiều điểm hỏi đáp ~~~~~~~~~~~~~~~~~~~

~~~~~~~~~~~ Và chúc các bạn trả lời câu hỏi này kiếm được nhiều k hơn ~~~~~~~~~~~~

Bình luận (0)
tran minh triet
Xem chi tiết
Trang Sún
4 tháng 4 2015 lúc 11:09

ta thấy : 2^2=4=4.1^2

             4^2=16=4.2^2

             6^2=36=4.3^2

              ...

             20^2=400=4.10^2

 =>S=2^2+4^2+6^2+...+20^2=4.(1^2+2^2+3^2+...+10^2)

                                         =4.385

                                         =1540

  Vậy tổng S = 1540

Bình luận (0)
nguyen truong giang
Xem chi tiết
Nguyễn Ngọc Quý
13 tháng 9 2015 lúc 10:18

\(S=2^2+4^2+.....+20^2\)

\(S=1^2.2^2+2^2.2^2+.......+10^2.2^2\)

\(S=2^2.\left(1^2+2^2+.....+10^2\right)\)

\(S=4.385=1540\) (đề bài)

Bình luận (0)
Harry Potter
Xem chi tiết
Hoàng Thị Ngọc Anh
2 tháng 1 2018 lúc 17:42

Câu hỏi tương tự: Câu hỏi của cát phượng - Toán lớp 7 | Học trực tuyến

Bình luận (0)
Harry Potter
2 tháng 1 2018 lúc 17:45

S = 22+ 42+...+202

⇔S = 22. (1+22+...+102)

⇔S = 4. 385

⇔S= 1540

Bình luận (1)
le thai quy
Xem chi tiết
robert lewandoski
3 tháng 7 2015 lúc 16:00

\(2^2+4^2+6^2+..+20^2=\left(1.2\right)^2+\left(2.2\right)^2+\left(2.3\right)^2+..+\left(2.10\right)^2\)

\(=1^2.2^2+2^2.2^2+2^2.3^2+..+2^2.10^2\)

\(=2^2.\left(1^2+2^2+3^2+..+10^2\right)=4.385=1540\)

bài này gặp nhiều rồi

Bình luận (0)
Lâm Bình
Xem chi tiết
Nguyễn Ngọc Anh Minh
2 tháng 7 2015 lúc 15:44

Ta có:

\(2^2\left(1^2+2^2+3^2+...+10^2\right)=2^2+4^2+6^2+...+20^2=S\)

=> \(S=2^2.385=1540\)

Bình luận (0)
Sếp Việt Đẹp Trai
13 tháng 9 2016 lúc 17:27

S=2^2+4^2+6^2+....+20^2=1540

Bình luận (0)
Mạc Vũ Phương
13 tháng 9 2017 lúc 21:17

Đặt A = 1^2+2^2+...+10^2 = 385

-> A x 2^2 =1^2 x 2^2 + 2^2 x 2^2 +....+ 10^2 x 2^2 = 385 x 2^2

-> A x 2^2 = (1x2)^2 + (2x2)^2 +...+ (10x2)^2 = 385 x 2^2

-> A x 2^2 = 2^2 + 4^2 +...+ 20^2 = 385 x 2^2

-> A x 2^2 = S = 385 x 2^2 = 385 x 4 =  1540

Bình luận (0)
Thiên Thu Nguyệt
Xem chi tiết
Hồ Ngọc Minh Châu Võ
16 tháng 7 2015 lúc 11:41

\(S=2^2+4^2+....+20^2=?\)

\(=\left(2.1\right)^2+\left(2.2\right)^2+\left(2.3\right)^2+....+\left(2.10\right)^2\)

\(=2^2.1^2+2^2.2^2+2^2.2^3+...+2^2.10^2\)

\(=2^2.\left(1^2+2^2+3^2+...+10^2\right)\)

\(=2^2.385\)

\(=4.385\)

\(=1540\)

Bình luận (0)
Hồ Ngọc Minh Châu Võ
16 tháng 7 2015 lúc 11:32

S=22+42+...+202

=> 1/2 .S=12+22+...+102

=> 1/2 .S=385

=> S = 385 . 2

=> S = 770

Bình luận (0)
vu ba hao
30 tháng 9 2016 lúc 13:07

P=(1.3)2+...+(10.3)

   =32(12+...+102)

   =9.385 = 3465

Bình luận (0)
Mary Dang
Xem chi tiết
nguyen minh nghia
15 tháng 7 2015 lúc 9:31

S=22+42+...+202

=> 1/2 .S=12+22+...+102

=> 1/2 .S=385

=> S = 385 . 2

=> S = 770

Bình luận (0)
Minh Hiền
15 tháng 7 2015 lúc 9:29

S=22+42+...+202

=> 1/2 .S=12+22+...+102

=> 1/2 .S=385

=> S = 385 . 2

=> S = 770

Bình luận (0)
Hồ Ngọc Minh Châu Võ
16 tháng 7 2015 lúc 11:54

\(S=2^2+4^2+6^2+...+20^2=?\)

\(=\left(2.1\right)^2+\left(2.2\right)^2+\left(2.3\right)^2...+\left(2.10\right)^2\)

\(=2^2.1^2+2^2.2^2+2^2.3^2+...+2^2.10^2\)

\(=2^2.\left(1^2+2^2+3^2+...+10^2\right)\)

\(=2^2.385\)

\(=4.385\)

\(=1540\)

câu đầu mà sai là dẫn cả bài toán sai theo

Bình luận (0)